Advanced Calculus Fitzpatrick 2e

Click For Summary
The discussion revolves around a request for specific homework questions from the book "Advanced Calculus 2e" by Fitzpatrick, specifically questions 14, 15, and 16 on page 20. The questions focus on proving inequalities using Cauchy's Inequality, emphasizing the nonnegativity of real numbers. Participants highlight the importance of having textbooks on hand at the beginning of classes to avoid last-minute issues with homework assignments. The conversation also acknowledges the unusual nature of asking for questions instead of answers, with one user commending another for their proactive approach. Overall, the thread underscores the challenges of accessing educational materials in a timely manner.
propensity
Messages
2
Reaction score
0
If anyone of you got the book Advanced Calculus 2e by Fitzpatrick I would appreciate your posting of the questions 14, 15, 16 on page 20. My order of the book hasn't reached me but I need to turn in the homework tomorrow. I just need the questions.

Thank you.
 
Physics news on Phys.org
Here are the questions:

14.(Cauchy’s Inequality) Using the fact that the square of a real number is nonnegative, prove that for any numbers a and b
Prove that:
ab \leq \frac{1}{2} \left( a^2+b^2)

15. Use Cauchy's Inequality to prove that if a \geq 0 and b \geq 0 , then
\sqrt{ab} \leq \frac{1}{2} \left( a + b \right)

16. use Cauchy's Inequality to prove for any numbers a and b and a natural number n

ab \leq \frac{1}{2} \left( na^2 + \frac{1}{n}b^2 \right)This is why you should buy your books at the bookstore for the beginning of classes(in case professors assign homework problems), even if you pre-order then online. You can always return them for a refund.
 
Wow, this is a first. I don't think I've seen anyone ask for a question rather than an answer. Good job rising to the occasion konthelion! And good advice.
 
Thanks konthelion for the reply and advice.
 
Question: A clock's minute hand has length 4 and its hour hand has length 3. What is the distance between the tips at the moment when it is increasing most rapidly?(Putnam Exam Question) Answer: Making assumption that both the hands moves at constant angular velocities, the answer is ## \sqrt{7} .## But don't you think this assumption is somewhat doubtful and wrong?

Similar threads

Replies
7
Views
1K
Replies
9
Views
4K
  • · Replies 57 ·
2
Replies
57
Views
5K
  • · Replies 1 ·
Replies
1
Views
4K
  • · Replies 5 ·
Replies
5
Views
2K
  • · Replies 16 ·
Replies
16
Views
5K
  • · Replies 11 ·
Replies
11
Views
4K
Replies
8
Views
3K
  • · Replies 2 ·
Replies
2
Views
3K
  • · Replies 29 ·
Replies
29
Views
2K